Difference between revisions of "Mock AIME 1 Pre 2005 Problems/Problem 12"
m (→Solution) |
(→Solution) |
||
(2 intermediate revisions by 2 users not shown) | |||
Line 3: | Line 3: | ||
== Solution == | == Solution == | ||
+ | Let <math>\angle{AB'C'} = \theta</math>. By some angle chasing in <math>\triangle{AB'E}</math>, we find that <math>\angle{EAB'} = 90^{\circ} - 2 \theta</math>. Before we apply the law of sines, we're going to want to get everything in terms of <math>\sin \theta</math>, so note that <math>\sin \angle{EAB'} = \sin(90^{\circ} - 2 \theta) = \cos 2 \theta = 1 - 2 \sin^2 \theta</math>. Now, we use law of sines, which gives us the following: | ||
+ | <math>\frac{\sin \theta}{5}=\frac{1 - 2 \sin^2 \theta}{23} \implies \sin \theta = \frac{-23 \pm 27}{20}</math>, | ||
+ | but since <math>\theta < 180^{\circ}</math>, we go with the positive solution. Thus, <math>\sin \theta = \frac15</math>. | ||
+ | |||
+ | Denote the intersection of <math>B'C'</math> and <math>AE</math> with <math>G</math>. By another application of the law of sines, <math>B'G = \frac{23}{\sqrt{24}}</math> and <math>AE = 10\sqrt{6}</math>. Since <math>\sin \theta = \frac15, GE = \frac{115}{\sqrt{24}}</math>, and <math>AG = AE - GE = 10\sqrt{6} - \frac{115}{\sqrt{24}} = \frac{5}{\sqrt{24}}</math>. Note that <math>\triangle{EB'G} \sim \triangle{C'AG}</math>, so <math>\frac{EG}{B'G}=\frac{C'G}{AG} \implies C'G = \frac{25}{\sqrt{24}}</math>. | ||
+ | |||
+ | Now we have that <math>AB = AE + EB = 10\sqrt{6} + 23</math>, and <math>B'C' = BC = B'G + C'G = \frac{23}{\sqrt{24}} + \frac{25}{\sqrt{24}} = \frac{48}{\sqrt{24}}=4\sqrt{6}</math>. Thus, the area of <math>ABCD</math> is <math>(10\sqrt{6} + 23)(4\sqrt{6}) = 92\sqrt{6} + 240</math>, and our final answer is <math>92 + 6 + 240 = \boxed{338}</math>. | ||
== See also == | == See also == | ||
Line 9: | Line 16: | ||
− | [[Category:Intermediate | + | [[Category:Intermediate Geometry Problems]] |
Latest revision as of 21:41, 26 December 2016
Problem
is a rectangular sheet of paper. and are points on and respectively such that . If is folded over , maps to on and maps to such that . If and , then the area of can be expressed as square units, where and are integers and is not divisible by the square of any prime. Compute .
Solution
Let . By some angle chasing in , we find that . Before we apply the law of sines, we're going to want to get everything in terms of , so note that . Now, we use law of sines, which gives us the following:
, but since , we go with the positive solution. Thus, .
Denote the intersection of and with . By another application of the law of sines, and . Since , and . Note that , so .
Now we have that , and . Thus, the area of is , and our final answer is .
See also
Mock AIME 1 Pre 2005 (Problems, Source) | ||
Preceded by Problem 11 |
Followed by Problem 13 | |
1 • 2 • 3 • 4 • 5 • 6 • 7 • 8 • 9 • 10 • 11 • 12 • 13 • 14 • 15 |